2.12.8  Zur Stetigkeit der Umkehrfunktion.

Der nächste Satz ist gewissermaßen eine Umkehrung der Aussage des Korollars 2.12.24 für monotone Funktionen.

Theorem 2.12.27. Es sei f : [a,b] eine monoton wachsende Funktion, welche alle Werte y [f(a),f(b)] annimmt. Dann ist f in [a,b] stetig.

Beweis. Die Funktion f ist monoton und durch f(a) und f(b) von unten bzw. oben beschränkt. Nach Satz 2.10.20 sowie Aufgabe 2.10.21 existieren dann die Grenzwerte f(x ± 0) für alle x ]a,b[ sowie am Rand die Grenzwerte f(a + 0) und f(b 0).

Es sei nun f(x 0)f(x + 0). Man betrachte zwei beliebige Zahlen x,x [a,b] mit x < x < x sowie Folgen {xk} k=1 mit a < xk < x und xk x als auch {xk} k=1 mit b > xk > x und xk x. Dann gilt x < x k1 < x und x < xk2 < x für alle genügend grosse k1,k2 N, und damit auch

f(x) f(x k1) f(x) f(x k2) f(x),k 1,k2 N. (2.77)

Geht man hier zum Grenzwert k1 und danach zu k2 über, so erhält man wegen f(x 0)f(x + 0)

f(x) f(x 0) < f(x + 0) f(x).

Damit nimmt f keine Funktionswerte in ]f(x 0),f(x + 0)[\{f(x)} an, was der Voraussetzung des Satzes widerspricht, also gilt f(x 0) = f(x + 0). Aus (2.77) folgt aber im Grenzübergang ebenso

f(x 0) f(x) f(x + 0)

und damit f(x) = f(x ± 0). Die Stetigkeit in den Endpunkten des Intervalls erhält man auf analogem Weg. □

Satz 2.12.27 lässt sich natürlich sofort auf monoton fallende Funktionen ausweiten. Man ersetze dazu f durch f.

Theorem 2.12.28. Die Funktion f bilde [a,b] bijektiv und monoton auf [α,β] ab. Dann ist die Umkehrfunktion f1 stetig auf [α,β].

Beweis. Wir wenden Satz 2.12.27 auf die Funktion f1 an. Diese ist zusammen mit f monoton und nimmt alle Werte des Intervalls [a,b] an. □

Aus Satz 2.12.28 folgt die (stückweise) Stetigkeit der Umkehrfunktionen der üblichen Elementarfunktionen, wie z.B. x, ln x, arcsin x.

Problem 2.12.29. Untersuchen Sie die Elementarfunktionen und deren Umkehrfunktionen auf Stetigkeit!

Remark 2.12.30. Wir merken an dieser Stelle an, dass die Umkehrfunktion einer stetigen Funktion im Allgemeinen nicht stetig ist. Als Beispiel betrachten wir die Abbildung f : [0, 2π[ gegeben durch f(x) = eix. Als Restriktion von Kompositionen stetiger Funktionen ist f(x) = eix stetig. Das Bild von f ist der Einheitskreis in und f ist bijektiv. Die Umkehrfunktion f1(z) erleidet andererseits einen Sprung von 2π wenn |z| = 1vom vierten in den ersten Quadranten wechselt und ist damit nicht stetig von nach [0, 2π[.